1 Introduction

In the past few years, many q-congruences and q-supercongruences have been established by different authors. See, for example, [2, 4,5,6,7,8, 11,12,13,14,15,16,17,18,19,20,21,22,23,24,25]. In particular, Guo and Schlosser [9, Theorem 5.5] proved that, for any odd integer \(n>3\),

$$\begin{aligned} \sum _{k=0}^{(n+1)/2}[4k+1]\frac{(q^{-1};q^2)_k^2(q;q^2)_k^2}{(q^4;q^2)_k^2(q^2;q^2)_k^2}q^{4k} \equiv 0 \pmod {[n]\Phi _n(q)^2}, \end{aligned}$$
(1.1)

where the q-shifted factorial is defined by

$$\begin{aligned} (a;q)_\infty =\prod _{j \geqslant 0}(1-aq^j) \quad \mathrm {and} \quad (a;q)_k = \frac{(a;q)_\infty }{(aq^k;q)_\infty }. \end{aligned}$$

For convenience, we adopt the notation

$$\begin{aligned} (a_1,a_2 ,\ldots ,a_r;q)_k=(a_1;q)_k(a_2;q)_k \cdots (a_r;q)_k, \quad k \in \mathbb {C} \cup \infty . \end{aligned}$$

The q-integer is defined by \([n]=[n]_q=(1-q^n)/(1-q)=1+q+\cdots +q^{n-1}\). Moreover, \(\Phi _n(q)\) represents the nth cyclotomic polynomial in q, which may be defined as

$$\begin{aligned} \Phi _n(q)=\prod _{\begin{array}{c} 1\leqslant k\leqslant n\\ \gcd (n,k)=1 \end{array}}(q-\zeta ^k) \end{aligned}$$

with \(\zeta \) being an nth primitive root of unity.

Guo and Schlosser [9, Conjecture 5.6] also proposed the following conjecture: for any odd integer \(n>3\),

$$\begin{aligned} \sum _{k=0}^{(n+1)/2}[4k+1]\frac{(aq^{-1};q^2)_k(q^{-1}/a;q^2)_k(q;q^2)_k^2}{(aq^4;q^2)_k(q^4/a;q^2)_k(q^2;q^2)_k^2}q^{4k}\equiv 0 \pmod {[n]\Phi _n(q)(1-aq^n)(a-q^n)}. \end{aligned}$$
(1.2)

It is known from [9] that, letting \(a \rightarrow 1\) in (1.2), we are led to

$$\begin{aligned} \sum _{k=0}^{(n+1)/2}[4k+1]\frac{(q^{-1};q^2)_k^2(q;q^2)_k^2}{(q^4;q^2)_k^2(q^2;q^2)_k^2}q^{4k} \equiv 0 \pmod {[n]\Phi _n(q)^3}, \end{aligned}$$

which is a refinement of the q-supercongruence (1.1).

Additionally, Guo and Schlosser [9, Conjecture 5.8] provided another similar conjecture: for any odd integer \(n>3\),

$$\begin{aligned}&\sum _{k=0}^{(n+1)/2}[4k+1]\frac{(aq^{-1};q^2)_k(q^{-1}/a;q^2)_k(q^{-1},q;q^2)_k}{(aq^4;q^2)_k(q^4/a;q^2)_k(q^4,q^2;q^2)_k}q^{6k}\equiv 0 \pmod {\Phi _n(q)^2(1-aq^n)(a-q^n)}. \end{aligned}$$
(1.3)

Likewise, when \(a \rightarrow 1\) in (1.3), we obtain

$$\begin{aligned} \sum _{k=0}^{(n+1)/2}[4k+1]\frac{(q^{-1};q^2)_k^3(q;q^2)_k}{(q^4;q^2)_k^3(q^2;q^2)_k}q^{6k} \equiv 0 \pmod {\Phi _n(q)^4}. \end{aligned}$$

Inspired by Guo and Schlosser’s work, we shall prove these two conjectures in this paper. Our proof relies on the following theorem, which may be deemed as a generalization of the q-supercongruences (1.2) and (1.3).

Theorem 1

Let \(n>3\) be an odd integer and \(r\in \{0,2\}\). Then, modulo \(\Phi _n(q)^2(1-aq^n)(a-q^n)\), we have

$$\begin{aligned}&\sum _{k=0}^{(n+1)/2}[4k+1]\frac{(aq^{-1},q^{-1}/a,q^{1-r},cq,dq,q;q^{2})_k}{(q^4/a,aq^4,q^{2+r},q^2/c,q^2/d,q^2;q^{2})_k}\bigg (\frac{q^{5+r}}{cd}\bigg )^k\nonumber \\&\qquad \equiv [n]Z_q(a,n)\frac{(aq^{-1},q^{-1}/a,q^{1-r},q/cd;q^2)_{(r+4)/2}}{(q,q^{r+2};q^2)_2(q^2/c,q^2/d;q^2)_{(r+4)/2}}\nonumber \\&\qquad \times \sum _{k=0}^{\frac{n-r-3}{2}}\frac{(aq^{r+3},q^{r+3}/a,q^5,q^{r+5}/cd;q^{2})_k}{(q^2,q^{6+r},q^{6+r}/c,q^{6+r}/d;q^{2})_k}q^{2k}, \end{aligned}$$
(1.4)

where

$$\begin{aligned} Z_q(a,n)&=(-1)^{\frac{r+2}{2}}q^{\frac{-2n+r^2+10r+26}{4}}\Bigg \{\frac{n(1-aq^2)(1-q^2/a)(1-a)a^{(n-1)/2}}{(aq,q/a;q^2)_{(r+2)/2}(1-a^n)}-\frac{(1-q^2)^2}{(q;q^2)_{(r+2)/2}^2}\Bigg \}\\&\quad \times \frac{(1-aq^n)(a-q^n)}{(1-a)^2}+q^{\frac{(r-1)(n+1)}{2}+r+8}\frac{(q^{n-2};q^4)_{2}}{(q^{n-r-1};q^2)_{r+2}}. \end{aligned}$$

Further, for the \(r=0\) and \(a \rightarrow 1\) case of Theorem 1, we have the following stronger conclusion.

Theorem 2

Let \(n>3\) be an odd integer. Then

$$\begin{aligned}&\sum _{k=0}^{M}[4k+1]\frac{(q^{-1};q^2)_k^2(q;q^2)_k^2(cq,dq;q^{2})_k}{(q^{4};q^2)_k^2(q^2;q^2)_k^2(q^2/c,q^2/d;q^{2})_k}\bigg (\frac{q^{5}}{cd}\bigg )^k\nonumber \\&\quad \equiv S_q(n)\frac{(q^{-1};q^2)_2^2(q/cd;q^2)_{2}}{(q^{2},q^2/c,q^2/d;q^2)_{2}}\nonumber \\&\quad \times \sum _{k=0}^{\frac{n-3}{2}}\frac{(q^{3};q^2)_k^2(q^{5},q^5/cd;q^{2})_k}{(q^2,q^6,q^6/c,q^6/d;q^{2})_k}q^{2k} \pmod {[n]\Phi _n(q)^3}, \end{aligned}$$
(1.5)

where \(M \in \{(n+1)/2,n-2\}\) and

$$\begin{aligned} S_q(n)=&\, q^{\frac{13-n}{2}}\frac{n^2(1-q^2)^2-(1+24q+22q^2+24q^3+q^4)}{24}[n]^3\\&+q^{\frac{15-n}{2}}\frac{[n-2][n+2]}{[n-1][n+1]}[n]. \end{aligned}$$

Putting \(c \rightarrow 1\) and \(d \rightarrow \infty \) in Theorem 2, we obtain the following result.

Corollary 3

Let \(n>3\) be an odd integer. Then, modulo \([n]\Phi _n(q)^3\),

$$\begin{aligned}&\sum _{k=0}^{M}(-1)^k[4k+1]\frac{(q^{-1};q^2)_k^2(q;q^2)_k^3}{(q^{4};q^2)_k^2(q^2;q^2)_k^3}q^{5k+k^2}\nonumber \\&\quad \equiv S_q(n)\frac{(q^{-1};q^2)_2^2}{(q^{2};q^2)_{2}^2} \sum _{k=0}^{\frac{n-3}{2}}\frac{(q^{3};q^2)_k^2(q^{5};q^{2})_k}{(q^2;q^2)_k(q^6;q^{2})_k^2}q^{2k}. \end{aligned}$$
(1.6)

When \(M=(p^l+1)/2\) and \(q \rightarrow 1\) in (1.6), we are led to the following: for odd prime \(p>3\),

$$\begin{aligned} \sum _{k=0}^{(p^l+1)/2}(-1)^k(4k+1)\frac{(-\frac{1}{2})_k^2(\frac{1}{2})_k^3}{k!^3(k+1)!^2}\equiv \frac{p^l}{4}\sum _{k=0}^{(p^l-3)/2}\frac{(\frac{3}{2})_k^2(\frac{5}{2})_k}{k!(k+2)!^2} \pmod {p^{l+3}}, \end{aligned}$$

where l is a positive integer, and the notation will be used frequently in this section.

Moreover, the case \(c \rightarrow 1\), \(d \rightarrow 1\) of Theorem 2 yields the following q-supercongruence.

Corollary 4

Let \(n>3\) be an odd integer. Then, modulo \([n]\Phi _n(q)^3\),

$$\begin{aligned}&\sum _{k=0}^{M}[4k+1]\frac{(q^{-1};q^2)_k^2(q;q^2)_k^4}{(q^{4};q^2)_k^2(q^2;q^2)_k^4}q^{5k}\nonumber \\&\quad \equiv S_q(n)\frac{(q^{-1};q^2)_2^2(q;q^2)_2}{(q^{2};q^2)_{2}^3}\sum _{k=0}^{\frac{n-3}{2}}\frac{(q^{3};q^2)_k^2(q^{5};q^{2})_k^2}{(q^2;q^2)_k(q^6;q^{2})_k^3}q^{2k}. \end{aligned}$$
(1.7)

Putting \(M=(p^l+1)/2\) and \(q \rightarrow 1\) in (1.7), we gain the following: for odd prime \(p>3\),

$$\begin{aligned} \sum _{k=0}^{(p^l+1)/2}(4k+1)\frac{(-\frac{1}{2})_k^2(\frac{1}{2})_k^4}{k!^4(k+1)!^2}\equiv \frac{3p^l}{16}\sum _{k=0}^{(p^l-3)/2}\frac{(\frac{3}{2})_k^2(\frac{5}{2})_k^2}{k!(k+2)!^3} \pmod {p^{l+3}}. \end{aligned}$$

In addition, letting \(r=2\) and \(a \rightarrow 1\) in Theorem 1, and applying the L’Hospital rule, we arrive at the following conclusion.

Theorem 5

Let \(n>3\) be an odd integer. Then

$$\begin{aligned}&\sum _{k=0}^{(n+1)/2}[4k+1]\frac{(q^{-1};q^2)_k^3(q,cq,dq;q^{2})_k}{(q^{4};q^2)_k^3(q^2,q^2/c,q^2/d;q^{2})_k}\bigg (\frac{q^{7}}{cd}\bigg )^k\nonumber \\&\quad \equiv T_q(n)\frac{(q^{-1};q^2)_3^3(q/cd;q^2)_{3}}{(q,q^{4};q^2)_2(q^2/c,q^2/d;q^2)_{3}}\nonumber \\&\quad \times \sum _{k=0}^{\frac{n-5}{2}}\frac{(q^{5};q^2)_k^3(q^7/cd;q^{2})_k}{(q^2,q^8,q^8/c,q^8/d;q^{2})_k}q^{2k}\pmod {\Phi _n(q)^4}, \end{aligned}$$
(1.8)

where

$$\begin{aligned} T_q(n)&=q^{\frac{25-n}{2}}\frac{(n^2-1)(1+q)^2(1-q^3)^2-24q(1+3q+7q^2+9q^3+7q^4+3q^5+q^6)}{24(1-q^3)^2(1+q+q^2)^2}[n]^3\\&\quad +q^{\frac{n+21}{2}}\frac{(q^{n-2};q^4)_{2}}{(q^{n-3};q^2)_4}[n]. \end{aligned}$$

Taking \(c \rightarrow q^{-2}\) and \(d \rightarrow \infty \) in Theorem 5, we obtain the following result.

Corollary 6

Let \(n>3\) be an odd integer. Then

$$\begin{aligned}&\sum _{k=0}^{(n+1)/2}(-1)^k[4k+1]\frac{(q^{-1};q^2)_k^4(q;q^2)_k}{(q^{4};q^2)_k^4(q^2;q^2)_k}q^{9k+k^2}\nonumber \\&\qquad \equiv T_q(n)\frac{(q^{-1};q^2)_3^3}{(q,q^4;q^2)_{2}(q^4;q^2)_3}\nonumber \\&\qquad \times \sum _{k=0}^{\frac{n-5}{2}}\frac{(q^{5};q^{2})_k^3}{(q^2,q^8,q^{10};q^{2})_k}q^{2k}\pmod {\Phi _n(q)^4}. \end{aligned}$$
(1.9)

Letting \(n=p^l\) be an odd prime power greater than 3 and \(q \rightarrow 1\) in (1.9), we get

$$\begin{aligned} \sum _{k=0}^{(p^l+1)/2}(-1)^k(4k+1)\frac{(-\frac{1}{2})_k^4(\frac{1}{2})_k}{k!(k+1)!^4} \equiv&\,\frac{p^l}{8}\sum _{k=0}^{(p^l-5)/2}\frac{(\frac{5}{2})_k^3}{k!(k+3)!(k+4)!} \pmod {p^{4}}. \end{aligned}$$

Likewise, putting \(c \rightarrow q^{-2}\) and \(d \rightarrow q^{-2}\) in Theorem 5, we get the following q-supercongruence.

Corollary 7

Let \(n>3\) be an odd integer. Then

$$\begin{aligned}&\sum _{k=0}^{(n+1)/2}[4k+1]\frac{(q^{-1};q^2)_k^5(q;q^2)_k}{(q^{4};q^2)_k^5(q^2;q^2)_k}q^{11k}\nonumber \\&\quad \equiv T_q(n)\frac{(q^{-1};q^2)_3^3(q^{5};q^2)_3}{(q,q^4;q^2)_{2}(q^4;q^2)_3^2}\nonumber \\&\quad \times \sum _{k=0}^{\frac{n-5}{2}}\frac{(q^{5};q^{2})_k^3(q^{11};q^2)_k}{(q^2,q^8;q^2)_k(q^{10};q^{2})_k^2}q^{2k}\pmod {\Phi _n(q)^4}. \end{aligned}$$
(1.10)

When \(n=p^r\) is an odd prime power greater than 3 and \(q \rightarrow 1\) in (1.10), we get

$$\begin{aligned} \sum _{k=0}^{(p^l+1)/2}(4k+1)\frac{(-\frac{1}{2})_k^5(\frac{1}{2})_k}{k!(k+1)!^5} \equiv&\frac{315p^l}{64}\sum _{k=0}^{(p^l-5)/2}\frac{(\frac{5}{2})_k^3(\frac{11}{2})_k}{k!(k+3)!(k+4)!^2} \pmod {p^{4}}. \end{aligned}$$

The rest of this paper is arranged as follows. Firstly, we present some lemmas which will be needed in the proof of Theorem 1 in Sect. 2. Then, we prove Theorem 1 and Guo and Schlosser’s conjectures (1.2) and (1.3) in Sect. 3. Our proof makes use of the ‘creative microscoping’ method which was recently introduced by Guo and Zudilin [10], and the Chinese remainder theorem for coprime polynomials. Finally, in Sect. 4, we give a proof of Theorem 2.

2 Some Lemmas

We shall make use of Watson’s \(_8\phi _7\) transformation [1, Appendix (III.18)], which can be expressed as

$$\begin{aligned}&_8\phi _7 \left[ \begin{array}{l} a,\,\,\,\,\, qa^{\frac{1}{2}},\,\,\,\,\,\, \,-qa^{\frac{1}{2}},\,\,\,\,\,\, \,\,\,b, \,\,\,\,\,\,\,\,\,\,\,\,\,\,\,\,\,c,\,\,\,\,\,\,\,\,\,\,\,\,\,\,\,\,\,d,\,\,\,\,\,\,\,\,\,\,\,\,\,\,\,\,\,e\,\,\,\,\,\,\,\,\,\,\,\,\,\,\,\,\,\,q^{-N} \\ \,\,\,\,\,\,\,\,\,\,\,\,\,a^{\frac{1}{2}},\,\,\,\,\,\,\,\, -a^{\frac{1}{2}},\,\,\,\,\,\,\,\,a q/b,\,\,\,\,\,\,\,\,a q/c,\,\,\,\,\,\,\,\,a q/d, \,\,\,\,\,\,\,\,aq/e, \,\,\,\,\,\,\,\,aq^{N+1}\end{array}; q,\,\frac{a^2q^{N+2}}{bcde} \right] \nonumber \\&\quad =\frac{(aq,aq/de;q)_{N}}{(aq/d,aq/e;q)_N}{_4\phi _3} \left[ \begin{array}{l} aq/bc,\,\, \,\,\,\, d,\,\,\,\,\,\,\, \,\,\,\,\,e,\,\,\,\,\,\,\,\,\,\,\,\, q^{-N} \\ \,\,\,\,\,\,\,\,\,\,\,\,\,\,\,\, aq/b,\,\,\, aq/c,\,\,\,deq^{-N}/a\, \end{array}; q,\,q \right] , \end{aligned}$$
(2.1)

where the basic hypergeometric series \(_{s+1}\phi _s\) is defined as

$$\begin{aligned} _{s+1}\phi _{s}\left[ \begin{array}{c} a_1,a_2,\ldots ,a_{s+1}\\ b_1,b_2,\ldots ,b_{s} \end{array};q,\, z \right] =\sum _{k=0}^{\infty }\frac{(a_1,a_2,\ldots , a_{s+1};q)_k z^k}{(q,b_1,\ldots ,b_{s};q)_k} \end{aligned}$$

with \(0<|z|<1\).

In this section, we shall give three lemmas, which will play an important role in our proof of Theorem 1.

Lemma 1

Let \(n>3\) be an odd integer and \(r\in \{0,2\}\). Then

$$\begin{aligned} \sum _{k=0}^{M}[4k+1]\frac{(aq^{-1},q^{-1}/a,q^{1-r}/b,cq,dq,q;q^{2})_k}{(q^4/a,aq^4,bq^{2+r},q^2/c,q^2/d,q^2;q^{2})_k}\bigg (\frac{bq^{5+r}}{cd}\bigg )^k \equiv 0 \pmod {\Phi _n(q)}, \end{aligned}$$
(2.2)

where \(M \in \{(n+1)/2,n-2\}\).

Proof

Let \(c_q(k)\) be the kth term on left-hand side of (2.2), i.e.,

$$\begin{aligned} c_q(k)=[4k+1]\frac{(aq^{-1},q^{-1}/a,q^{1-r}/b,cq,dq,q;q^{2})_k}{(q^4/a,aq^4,bq^{2+r},q^2/c,q^2/d,q^2;q^{2})_k}\bigg (\frac{bq^{5+r}}{cd}\bigg )^k. \end{aligned}$$

Using the following q-congruence due to Guo and Schlosser [9, Lemma 3.1]

$$\begin{aligned}&\frac{(aq;q^2)_{(n-1)/2-k}}{(q^2/a;q^2)_{(n-1)/2-k}}\\&\quad \equiv (-a)^{\frac{n-1}{2}-2k}q^{\frac{(n-1)^2}{4}+k}\frac{(aq;q^2)_k}{(q^2/a;q^2)_k} \pmod {\Phi _n(q)}, \end{aligned}$$

we have

$$\begin{aligned} c_q(k) \equiv -c_q((n-1)/2-k) \pmod {\Phi _n(q)}. \end{aligned}$$

This proves that

$$\begin{aligned} \begin{aligned}&\sum _{k=0}^{(n-1)/2}[4k+1]\frac{(aq^{-1},q^{-1}/a,q^{1-r}/b,cq,dq,q;q^{2})_k}{(q^4/a,aq^4,bq^{2+r},q^2/c,q^2/d,q^2;q^{d})_k}\bigg (\frac{bq^{5+r}}{cd}\bigg )^k \equiv 0\quad \pmod {\Phi _n(q)}. \end{aligned} \end{aligned}$$
(2.3)

Since the numerator of \(c_q(k)\) contains the factor \((q;q^2)_k\), we see that \(c_q(k)\) is congruent to 0 modulo \(\Phi _n(q)\) for \((n+1)/2\le k\le n-2\). Thus, the proof of Lemma 1 is completed. \(\square \)

Lemma 2

Let \(n>3\) be an odd integer and \(r\in \{0,2\}\). Then

$$\begin{aligned} \begin{aligned}&\sum _{k=0}^{M}[4k+1]\frac{(aq^{-1},q^{-1}/a,q^{1-r}/b,cq,dq,q;q^{2})_k}{(q^4/a,aq^4,bq^{2+r},q^2/c,q^2/d,q^2;q^{2})_k}\bigg (\frac{bq^{5+r}}{cd}\bigg )^k\\&\quad \equiv [n]b^{\frac{n+1}{2}}q^{\frac{(r-1)(n+1)}{2}}\frac{(1-q^{n-2})(1-q^{n+2})(q^{-2-r}/b;q^2)_{(n+1)/2}}{(1-q^{-3})(1-q^{-1})(bq^{2+r};q^2)_{(n+1)/2}}\\&\qquad \times \sum _{k=0}^{(n+1)/2}\frac{(aq^{-1},q^{-1}/a,q^{1-r}/b,q/cd;q^{2})_k}{(q^2,q^2/c,q^2/d,q^{-2-r}/b;q^{2})_k}q^{2k} \pmod {\Phi _n(q)(1-aq^n)(a-q^n)}, \end{aligned} \end{aligned}$$
(2.4)

where \(M \in \{(n+1)/2,n-2\}\).

Proof

When \(a=q^{-n}\) or \(q^n\), the left-hand side of (2.4) equals

$$\begin{aligned}&_8\phi _7 \left[ \begin{array}{l} q,\,\,\,\,\,\,\, q^{\frac{5}{2}},\,\,\,\,\,\,\,\, \,-q^{\frac{5}{2}},\,\,\,\,\,\, \,\,\,cq, \,\,\,\,\,\,\,\,\,\,\,\,\,dq,\,\,\,\,\,\,\,\,\,\,\,\,\,q^{1-r}/b,\,\,\,\,\,\,q^{-1-n},\,\,\,\,\,\,\,\,\,q^{-1+n} \\ \,\,\,\,\,\,\,\,\,\,\,\,\,q^{\frac{1}{2}},\,\,\,\,\,\,\,\, -q^{\frac{1}{2}},\,\,\,\,\,\,\,\,q^2/c,\,\,\,\,\,\,\,\,q^2/d,\,\,\,\,\,\,\,\,\,\,bq^{2+r}, \,\,\,\,\,\,\,\,\,\,q^{4+n}, \,\,\,\,\,\,\,\,\,\,\,\,q^{4-n}\end{array}; q^2,\,\frac{bq^{5+r}}{cd} \right] . \end{aligned}$$

Applying Watson’s \(_8\phi _7\) transformation (2.1), we obtain

$$\begin{aligned}&\sum _{k=0}^{M}[4k+1]\frac{(q^{-1+n},q^{-1-n},q^{1-r}/b,cq,dq,q;q^{2})_k}{(q^{4-n},aq^{4+n},bq^{2+r},q^2/c,q^2/d,q^2;q^{2})_k}\bigg (\frac{bq^{5+r}}{cd}\bigg )^k\\&\quad =\frac{(q^3,bq^{3+r-n};q^2)_{(n+1)/2}}{(bq^{2+r},q^{4-n};q^2)_{(n+1)/2}}\sum _{k=0}^{(n+1)/2}\frac{(q^{-1+n},q^{-1-n},q^{1-r}/b,q/cd;q^{2})_k}{(q^2,q^2/c,q^2/d,q^{-2-r}/b;q^{2})_k}q^{2k} \\&\quad =[n]b^{\frac{n+1}{2}}\frac{(1-q^{n-2})(1-q^{n+2})(q^{-2-r}/b;q^2)_{(n+1)/2}}{(1-q^{-3})(1-q^{-1})(bq^{2+r};q^2)_{(n+1)/2}}q^{\frac{(r-1)(n+1)}{2}}\\ \nonumber&\qquad \times \sum _{k=0}^{(n+1)/2}\frac{(q^{-1+n},q^{-1-n},q^{1-r}/b,q/cd;q^{2})_k}{(q^2,q^2/c,q^2/d,q^{-2-r}/b;q^{2})_k}q^{2k}, \end{aligned}$$

which means that the q-supercongruence (2.4) holds modulo \(1-aq^n\) and \(a-q^n\). The proof then follows from Lemma 1 and the fact that \(\Phi _n(q)\), \(1-aq^n\) and \(a-q^n\) are pairwise relatively prime polynomials. \(\square \)

Lemma 3

Let \(n>3\) be an odd integer and \(r\in \{0,2\}\). Then

$$\begin{aligned} \begin{aligned}&\sum _{k=0}^{M}[4k+1]\frac{(aq^{-1},q^{-1}/a,q^{1-r}/b,cq,dq,q;q^{2})_k}{(q^4/a,aq^4,bq^{2+r},q^2/c,q^2/d,q^2;q^{2})_k}\bigg (\frac{bq^{5+r}}{cd}\bigg )^k\\&\quad \equiv [n]\frac{(bq^2;q^2)_{r/2}( b;q^2)_{(r+4)/2}(q;q^2)_{(n-1)/2}^2}{(q;q^2)_{2}(q^4/a;q^2)_{(n+r-1)/2}(aq^4;q^2)_{(n+r-1)/2}}\\&\quad \times \sum _{k=0}^{(n+r-1)/2}\frac{(aq^{-1},q^{-1}/a,q^{1-r}/b,q/cd;q^{2})_k}{(q^2,q^2/c,q^2/d,q^{-2-r}/b;q^{2})_k}q^{2k} \pmod {b-q^n}, \end{aligned} \end{aligned}$$

where \(M \in \{(n+1)/2,n-2\}\).

Proof

Letting \(b=q^n\) in the left-hand side of the above relation, we have

$$\begin{aligned}&\sum _{k=0}^{M}[4k+1]\frac{(aq^{-1},q^{-1}/a,q^{1-r-n},cq,dq,q;q^{2})_k}{(q^4/a,aq^4,q^{2+r+n},q^2/c,q^2/d,q^2;q^{2})_k}\bigg (\frac{q^{5+r+n}}{cd}\bigg )^k\\&\quad =\frac{[n](q^{2+n};q^2)_{r/2}( q^n;q^2)_{(r+4)/2}(q;q^2)_{(n-1)/2}^2}{(q;q^2)_{2}(q^4/a;q^2)_{(n+r-1)/2}(aq^4;q^2)_{(n+r-1)/2}}\\&\qquad \times \sum _{k=0}^{\frac{n+r-1}{2}}\frac{(aq^{-1},q^{-1}/a,q^{1-r-n},q/cd;q^{2})_k}{(q^2,q^2/c,q^2/d,q^{-2-r-n};q^{2})_k}q^{2k}, \end{aligned}$$

which follows from the substitutions \(a= q, q \mapsto q^2, b= cq, c = dq, d = aq^{-1}, e = q^{-1}/a\), and \(N = (n+r-1)/2\) in Watson’s \(_8\phi _7\) transformation (2.1). Namely, Lemma 3 is true. \(\square \)

3 Proof of Theorem 1

Firstly, we need the following two q-congruences:

$$\begin{aligned}&\frac{(b-q^n)(ab-1-a^2+aq^n)}{(a-b)(1-ab)} \equiv 1 \pmod {(1-aq^n)(a-q^n)}, \\&\frac{(1-aq^n)(a-q^n)}{(a-b)(1-ab)} \equiv 1 \pmod {(b-q^n)}, \end{aligned}$$

which can be found in Guo [3]. Employing the Chinese remainder theorem for coprime polynomials and combining Lemmas 2 and 3, we conclude that, modulo \(\Phi _n(q)(1-aq^n)(a-q^n)(b-q^n)\),

$$\begin{aligned}&\sum _{k=0}^{(n+1)/2}[4k+1]\frac{(aq^{-1},q^{-1}/a,q^{1-r}/b,cq,dq,q;q^{2})_k}{(q^4/a,aq^4,bq^{2+r},q^2/c,q^2/d,q^2;q^{2})_k}\bigg (\frac{bq^{5+r}}{cd}\bigg )^k\nonumber \\&\qquad \equiv [n]W_q(a,b,n)\sum _{k=0}^{\frac{n+1}{2}}\frac{(aq^{-1},q^{-1}/a,q^{1-r}/b,q/cd;q^{2})_k}{(q^2,q^2/c,q^2/d,q^{-2-r}/b;q^{2})_k}q^{2k}, \end{aligned}$$
(3.1)

where

$$\begin{aligned} W_q(a,b,n)&=b^{\frac{n+1}{2}}\frac{(1-q^{n-2})(1-q^{n+2})(q^{-2-r}/b;q^2)_{(n+1)/2}}{(1-q^{-3})(1-q^{-1})(bq^{2+r};q^2)_{(n+1)/2}}q^{\frac{(r-1)(n+1)}{2}}\\&\quad \times \frac{(b-q^n)(ab-1-a^2+aq^n)}{(a-b)(1-ab)}\\&\quad +\frac{(bq^2;q^2)_{r/2}( b;q^2)_{(r+4)/2}(q;q^2)_{(n-1)/2}^2}{(q;q^2)_{2}(q^4/a;q^2)_{(n+r-1)/2}(aq^4;q^2)_{(n+r-1)/2}} \frac{(1-aq^n)(a-q^n)}{(a-b)(1-ab)}. \end{aligned}$$

Obviously, the q-supercongruence (3.1) can be expressed as modulo \(\Phi _n(q)(1-aq^n)(a-q^n)(b-q^n)\),

$$\begin{aligned}&\sum _{k=0}^{(n+1)/2}[4k+1]\frac{(aq^{-1},q^{-1}/a,q^{1-r}/b,cq,dq,q;q^{2})_k}{(q^4/a,aq^4,bq^{2+r},q^2/c,q^2/d,q^2;q^{2})_k}\bigg (\frac{bq^{5+r}}{cd}\bigg )^k\nonumber \\&\quad \equiv [n] W_q(a,b,n) \sum _{k=0}^{(r+2)/2}\frac{(aq^{-1},q^{-1}/a,q^{1-r}/b,q/cd;q^{2})_k}{(q^2,q^2/c,q^2/d,q^{-2-r}/b;q^{2})_k}q^{2k}\nonumber \\&\qquad +[n] W_q(a,b,n)\sum _{k=0}^{(n-r-3)/2}\frac{(aq^{-1},q^{-1}/a,q^{1-r}/b,q/cd;q^{2})_{k+(r+4)/2}}{(q^2,q^2/c,q^2/d,q^{-2-r}/b;q^{2})_{k+(r+4)/2}}q^{2k+r+4} . \end{aligned}$$
(3.2)

Letting \(b \rightarrow 1\) in (3.2) and using the relation

$$\begin{aligned} (1-q^n)(1+a^2-a-aq^n)=(1-a)^2+(1-aq^n)(a-q^n), \end{aligned}$$

we attain modulo \(\Phi _n(q)(1-aq^n)(a-q^n)(b-q^n)\),

$$\begin{aligned}&\sum _{k=0}^{(n+1)/2}[4k+1]\frac{(aq^{-1},q^{-1}/a,q^{1-r},cq,dq,q;q^{2})_k}{(q^4/a,aq^4,q^{2+r},q^2/c,q^2/d,q^2;q^{2})_k}\bigg (\frac{q^{5+r}}{cd}\bigg )^k\nonumber \\&\quad \equiv [n] Y_q(a,n)\frac{(aq^{-1},q^{-1},q^{1-r},q/cd;q^2)_{(r+4)/2}}{(q,q^{r+2};q^2)_2(q^2/c,q^2/d;q^2)_{(r+4)/2}}q^{r+4}\nonumber \\&\qquad \times \sum _{k=0}^{(n-r-3)/2}\frac{(aq^{r+3},q^{r+3}/a,q^{1-r},q^{r+5}/cd;q^{5})_{k}}{(q^2,q^{6+r}/c,q^{6+r}/d,q^{6+r};q^{2})_{k}}q^{2k}, \end{aligned}$$
(3.3)

where

$$\begin{aligned} Y_q(a,n)&=\Bigg \{\frac{(q^{n-2};q^4)_2}{(q^{n-r-1};q^2)_{r+2}}q^{\frac{(r-1)(n+1)}{2}+4}+(-1)^{(r+2)/2}\frac{(q;q^2)_{(n-1)/2}^2}{(q^4/a,aq^4;q^2)_{(n+r-1)/2}} \Bigg \}\\&\quad \times \frac{(1-aq^n)(a-q^n)}{(1-a)^2}+\frac{(q^{n-2};q^4)_2}{(q^{n-r-1};q^2)_{r+2}}q^{\frac{(r-1)(n+1)}{2}+4}. \end{aligned}$$

Noting \(q^n \equiv 1 \pmod {\Phi _n(q)}\) and recalling

$$\begin{aligned}&(aq^2,q^2/a;q^2)_{(n-1)/2}\equiv (-1)^{(n-1)/2}\frac{(1-a^n)q^{-(n-1)^2/4}}{(1-a)a^{(n-1)/2}} \pmod {\Phi _n(q)},\\&(aq,q/a;q^2)_{(n-1)/2}\equiv (-1)^{(n-1)/2}\frac{(1-a^n)q^{(1-n^2)/4}}{(1-a)a^{(n-1)/2}} \pmod {\Phi _n(q)}, \end{aligned}$$

which were first observed by Guo [3, Lemma 2.1], we are led to the following q-congruence: modulo \(\Phi _n(q)\),

$$\begin{aligned}&\frac{(q^{n-2};q^4)_2}{(q^{n-r-1};q^2)_{r+2}}q^{\frac{(r-1)(n+1)}{2}+4}+(-1)^{(r+2)/2}\frac{(q;q^2)_{(n-1)/2}^2}{(q^4/a,aq^4;q^2)_{(n+r-1)/2}} \nonumber \\&\quad \equiv (-1)^{\frac{r+2}{2}}q^{\frac{-2n+r^2+6r+10}{4}}\Bigg \{\frac{n(1-aq^2)(1-q^2/a)(1-a)a^{(n-1)/2}}{(aq,q/a;q^2)_{(r+2)/2}(1-a^n)}-\frac{(1-q^2)^2}{(q;q^2)_{(r+2)/2}^2}\Bigg \}. \end{aligned}$$
(3.4)

The proof then follows from (3.3) and (3.4).

Obviously, Theorem 1 also holds true when the summation in the left-hand side of (1.4) is from 0 to \(n-2\).

We now prove (1.2) and (1.3) which were conjectured by Guo and Schlosser.

Proof of (1.2)

When \(cd=q\) and \(r=0\) in Theorem 1, we have

$$\begin{aligned} \sum _{k=0}^{(n+1)/2}[4k+1]\frac{(aq^{-1};q^2)_k(q^{-1}/a;q^2)_k(q;q^2)_k^2}{(aq^4;q^2)_k(q^4/a;q^2)_k(q^2;q^2)_k^2}q^{4k} \equiv 0 \pmod {\Phi _n(q)^2(1-aq^n)(a-q^n)}. \end{aligned}$$

It remains to show that

$$\begin{aligned} \sum _{k=0}^{(n+1)/2}[4k+1]\frac{(aq^{-1};q^2)_k(q^{-1}/a;q^2)_k(q;q^2)_k^2}{(aq^4;q^2)_k(q^4/a;q^2)_k(q^2;q^2)_k^2}q^{4k} \equiv 0 \pmod {[n]}. \end{aligned}$$
(3.5)

For \(n>3\), let \(\zeta \ne 1\) be an nth root of unity, not necessarily primitive. Then \(\zeta \) must be a primitive mth root of unity with m|n. Let \(\alpha _q(k)\) denote the kth term on the left-hand side of (3.5):

$$\begin{aligned} \alpha _q(k)=[4k+1]\frac{(aq^{-1};q^2)_k(q^{-1}/a;q^2)_k(q;q^2)_k^2}{(aq^4;q^2)_k(q^4/a;q^2)_k(q^2;q^2)_k^2}q^{4k}. \end{aligned}$$

Putting \(n=m\), \(cd=q\), \(r=0\) and \(b \rightarrow 1\) in Lemma 1, and since \(\alpha _\zeta (k)=0\) for \((m+1)/2<k \le m-1\), we are led to

$$\begin{aligned} \sum _{k=0}^{(m+1)/2}\alpha _\zeta (k)=\sum _{k=0}^{m-1}\alpha _\zeta (k)=0. \end{aligned}$$

Since

$$\begin{aligned} \frac{\alpha _\zeta (lm+k)}{\alpha _\zeta (lm)}=\lim _{q\rightarrow \zeta }\frac{\alpha _q(lm+k)}{\alpha _q(lm)}=\alpha _\zeta (k), \end{aligned}$$

we immediately obtain

$$\begin{aligned} \sum _{k=0}^{(n+1)/2}\alpha _\zeta (k)=\sum _{l=0}^{(n/m-3)/2}\alpha _\zeta (lm)\sum _{k=0}^{m-1}\alpha _\zeta (k)+\sum _{k=0}^{(m+1)/2}\alpha _\zeta ((n-m)/2+k)=0, \end{aligned}$$

which shows that the cyclotomic polynomial \(\Phi _m(q)\) divides the sum \(\sum _{k=0}^{(n+1)/2}\alpha _q(k)\). In view of

$$\begin{aligned} \prod _{m|n,m>1}\Phi _m(q)=[n], \end{aligned}$$

the proof of (3.5) is completed and therefore (1.2) is true. \(\square \)

Proof of (1.3)

Likewise, letting \(cd=q\) and \(r=2\) in Theorem 1, we get (1.3) immediately. \(\square \)

4 Proof of Theorem 2

Through the L’Hospital rule, we have

$$\begin{aligned}&\lim \limits _{a \rightarrow 1} \Bigg \{\frac{n(1-aq^2)(1-q^2/a)(1-a)a^{(n-1)/2}}{(1-aq)(1-q/a)(1-a^n)}-(1+q)^2\Bigg \} \times \frac{(1-aq^n)(a-q^n)}{(1-a)^2}\\&\quad =-\frac{n^2(1-q^2)^2-(1+24q+22q^2+24q^3+q^4)}{24}[n]^2. \end{aligned}$$

Hence, when \(r=0\) and \(a \rightarrow 1\) in Theorem 1, we know that (1.5) holds modulo \(\Phi _n(q)^4\). It remains to show that

$$\begin{aligned} \sum _{k=0}^{M}[4k+1]\frac{(q^{-1};q^2)_k^2(q;q^2)_k^2(cq,dq;q^{2})_k}{(q^4;q^2)_k^2(q^2;q^2)_k^2(q^2/c,q^2/d;q^2)_k}\bigg (\frac{q^{5}}{cd}\bigg )^k \equiv 0 \pmod {[n]}, \end{aligned}$$
(4.1)

where \(M \in \{(n+1)/2,n-2\}\). Just like the proof of (3.5), let \(\zeta \ne 1\) be a primitive mth root of unity with m|n and let \(\beta _q(k)\) be the kth term on the left-hand side of (4.1), i.e.,

$$\begin{aligned} \beta _q(k)=[4k+1]\frac{(q^{-1};q^2)_k^2(q;q^2)_k^2(cq,dq;q^{2})_k}{(q^4;q^2)_k^2(q^2;q^2)_k^2(q^2/c,q^2/d;q^2)_k}\bigg (\frac{q^{5}}{cd}\bigg )^k. \end{aligned}$$

Fixing \(n=m\), \(a \rightarrow 1\), \(b\rightarrow 1\), and \(x=0\) in Lemma 1, and noticing that \(\beta _\zeta (k)=0\) for \((m+1)/2<k \le m-1\), we have

$$\begin{aligned} \sum _{k=0}^{(m+1)/2}\beta _\zeta (k)=\sum _{k=0}^{m-1}\beta _\zeta (k)=0. \end{aligned}$$

Similarly as before, we get

$$\begin{aligned}&\sum _{k=0}^{n-2}\beta _\zeta (k)=\sum _{l=0}^{n/m-2}\sum _{k=0}^{m-1}\beta _\zeta (lm+k)+\sum _{k=0}^{m-2}\beta _\zeta ((n-m)+k)=0,\\&\sum _{k=0}^{(n+1)/2}\beta _\zeta (k)=\sum _{l=0}^{(n/m-3)/2}\beta _\zeta (lm)\sum _{k=0}^{m-1}\beta _\zeta (k)+\sum _{k=0}^{(m+1)/2}\beta _\zeta ((n-m)/2+k)=0, \end{aligned}$$

which means that the cyclotomic polynomial \(\Phi _m(q)\) divides the sums \(\sum _{k=0}^{(n+1)/2}\beta _q(k)\) and \(\sum _{k=0}^{n-2}\beta _q(k)\). Since

$$\begin{aligned} \prod _{m|n,m>1}\Phi _m(q)=[n], \end{aligned}$$

we immediately obtain (4.1). The q-supercongruence (1.5) then follows from the fact that [n] is coprime with the denominator of the right-hand side of (1.5).